Skip to main content

Advertisement

Log in

Extremals for fractional order Hardy–Sobolev–Maz’ya inequality

  • Published:
Calculus of Variations and Partial Differential Equations Aims and scope Submit manuscript

Abstract

In this article, we derive the existence of positive solution of a semi-linear, non-local elliptic PDE, involving a singular perturbation of the fractional laplacian, coming from the fractional Hardy–Sobolev–Maz’ya inequality, derived in this paper. We also derive symmetry properties and a precise asymptotic behaviour of solutions.

This is a preview of subscription content, log in via an institution to check access.

Access this article

Price excludes VAT (USA)
Tax calculation will be finalised during checkout.

Instant access to the full article PDF.

Similar content being viewed by others

References

  1. Abdellaoui, B., Medina, M., Peral, I., Primo, A.: The effect of the Hardy potential in some Calderón–Zygmund properties for the fractional Laplacian. J. Differ. Equ. 260(11), 8160–8206 (2016)

    Article  MATH  Google Scholar 

  2. Badiale, M., Tarantello, G.: A Sobolev–Hardy inequality with applications to a nonlinear elliptic equation arising in astrophysics. Arch. Ration. Mech. Anal. 163(4), 259–293 (2002)

    Article  MathSciNet  MATH  Google Scholar 

  3. Baernstein, A. II.: A unified approach to symmetrization. In: Partial Differential Equations of Elliptic Type (Cortona, 1992), Sympos. Math., XXXV, pp. 47–91. Cambridge Univ. Press, Cambridge (1994)

  4. Berestycki, H., Nirenberg, L.: On the method of moving planes and the sliding method. Bol. Soc. Brasil. Mat. (N.S.) 22(1), 1–37 (1991)

    Article  MathSciNet  MATH  Google Scholar 

  5. Bogdan, K., Dyda, B.: The best constant in a fractional Hardy inequality. Math. Nachr. 284(5–6), 629–638 (2011)

    Article  MathSciNet  MATH  Google Scholar 

  6. Brézis, H., Lieb, E.: A relation between pointwise convergence of functions and convergence of functionals. Proc. Am. Math. Soc. 88(3), 486–490 (1983)

    Article  MathSciNet  MATH  Google Scholar 

  7. Caffarelli, L., Silvestre, L.: An extension problem related to the fractional Laplacian. Commun. Partial Differ. Equ. 32(7–9), 1245–1260 (2007)

    Article  MathSciNet  MATH  Google Scholar 

  8. Castorina, D., Fabbri, I., Mancini, G., Sandeep, K.: Hardy–Sobolev inequalities and hyperbolic symmetry. Atti Accad. Naz. Lincei Rend. Lincei Mat. Appl. 19(3), 189–197 (2008)

    Article  MathSciNet  MATH  Google Scholar 

  9. Chen, W., Li, C., Biao, O.: Classification of solutions for an integral equation. Commun. Pure Appl. Math. 59(3), 330–343 (2006)

    Article  MathSciNet  MATH  Google Scholar 

  10. Di Nezza, E., Palatucci, G., Valdinoci, E.: Hitchhiker’s guide to the fractional Sobolev spaces. Bull. Sci. Math. 136(5), 521–573 (2012)

    Article  MathSciNet  MATH  Google Scholar 

  11. Dipierro, S., Montoro, L., Peral, I., Sciunzi, B.: Qualitative properties of positive solutions to nonlocal critical problems involving the Hardy–Leray potential. Calc. Var. Partial Differ. Equ. 55(4), Art. 99, 29 (2016)

    MathSciNet  MATH  Google Scholar 

  12. Dipierro, S., Valdinoci, E.: A density property for fractional weighted Sobolev spaces. Atti Accad. Naz. Lincei Rend. Lincei Mat. Appl. 26(4), 397–422 (2015)

    Article  MathSciNet  MATH  Google Scholar 

  13. Dyda, B., Lehrbäck, J., Vähäkangas, A.: Fractional Hardy–Sobolev type inequalities for half space and John domain (2017). arXiv: 1709.03296v1

  14. Fall, M.M., Weth, T.: Nonexistence results for a class of fractional elliptic boundary value problems. J. Funct. Anal. 263(8), 2205–2227 (2012)

    Article  MathSciNet  MATH  Google Scholar 

  15. Felmer, P., Wang, Y.: Radial symmetry of positive solutions to equations involving the fractional laplacian. Commun. Contemp. Math. 16(01), 1350023 (2014)

    Article  MathSciNet  MATH  Google Scholar 

  16. Frank, R.L., Lieb, E.H., Seiringer, R.: Hardy-Lieb-Thirring inequalities for fractional Schrödinger operators. J. Am. Math. Soc. 21(4), 925–950 (2008)

    Article  MATH  Google Scholar 

  17. Frank, R.L., Jin, T., Xiong, J.: Minimizers for the fractional Sobolev inequality on domains. J. Calc. Var. 57(3), 43 (2018)

    Article  MathSciNet  MATH  Google Scholar 

  18. Frank, R.L., Seiringer, R.: Non-linear ground state representations and sharp Hardy inequalities. J. Funct. Anal. 255(12), 3407–3430 (2008)

    Article  MathSciNet  MATH  Google Scholar 

  19. Frank, R.L., Seiringer, R..: Sharp fractional Hardy inequalities in half-spaces. In: Around the Research of Vladimir Maz’ya. I, vol 11 of Int. Math. Ser. (N. Y.), pp. 161–167. Springer, New York (2010)

    Google Scholar 

  20. Gazzini, M., Musina, R.: Hardy–Sobolev–Maz’ya inequalities: symmetry and breaking symmetry of extremal functions. Commun. Contemp. Math. 11(6), 993–1007 (2009)

    Article  MathSciNet  MATH  Google Scholar 

  21. Gazzini, M., Musina, R.: On a Sobolev-type inequality related to the weighted \(p\)-Laplace operator. J. Math. Anal. Appl. 352(1), 99–111 (2009)

    Article  MathSciNet  MATH  Google Scholar 

  22. Ghoussoub, N., Shakerian, S.: Borderline variational problems involving fractional Laplacians and critical singularities. Adv. Nonlinear Stud. 15(3), 527–555 (2015)

    Article  MathSciNet  MATH  Google Scholar 

  23. Herbst, I.W.: Spectral theory of the operator \((p^2 +m^2)^{1/2} -Ze^2/r\). Commun. Math. Phys. 53(3), 285–294 (1977)

    Article  MATH  MathSciNet  Google Scholar 

  24. Il’in, V.P.: Some integral inequalities and their applications in the theory of differentiable functions of several variables. Mat. Sb. (N.S.) 54(96), 331–380 (1961)

    MathSciNet  Google Scholar 

  25. Jin, T., Li, Y.Y., Xiong, J.: On a fractional Nirenberg problem, part I: blow up analysis and compactness of solutions. J. Eur. Math. Soc. (JEMS) 16(6), 1111–1171 (2014)

    Article  MathSciNet  MATH  Google Scholar 

  26. Kassmann, M.: A priori estimates for integro-differential operators with measurable kernels. Calc. Var. Partial Differ. Equ. 34(1), 1–21 (2009)

    Article  MathSciNet  MATH  Google Scholar 

  27. Lieb, E.H.: Sharp constants in the Hardy–Littlewood–Sobolev and related inequalities. Ann. Math. (2) 118(2), 349–374 (1983)

    Article  MathSciNet  MATH  Google Scholar 

  28. Lions, P.-L.: The concentration-compactness principle in the calculus of variations. The locally compact case. I. Ann. Inst. H. Poincaré Anal. Non Linéaire 1(2), 109–145 (1984)

    Article  MathSciNet  MATH  Google Scholar 

  29. Lions, P.-L.: The concentration-compactness principle in the calculus of variations. The locally compact case. II. Ann. Inst. H. Poincaré Anal. Non Linéaire 1(4), 223–283 (1984)

    Article  MathSciNet  MATH  Google Scholar 

  30. Li, Y.Y.: Remark on some conformally invariant integral equations: the method of moving spheres. J. Eur. Math. Soc. (JEMS) 6(2), 153–180 (2004)

    Article  MathSciNet  MATH  Google Scholar 

  31. Mancini, G., Fabbri, I., Sandeep, K.: Classification of solutions of a critical Hardy–Sobolev operator. J. Differ. Equ. 224(2), 258–276 (2006)

    Article  MathSciNet  MATH  Google Scholar 

  32. Mancini, G., Sandeep, K.: On a semilinear elliptic equation in \({\mathbb{H}}^n\). Ann. Sc. Norm. Super. Pisa Cl. Sci. (5) 7(4), 635–671 (2008)

    MathSciNet  MATH  Google Scholar 

  33. Maz’ja, V.G.: Sobolev spaces. Springer Series in Soviet Mathematics (Translated from the Russian by T. O. Shaposhnikova). Springer, Berlin (1985)

    Book  Google Scholar 

  34. Musina, R.: Ground state solutions of a critical problem involving cylindrical weights. Nonlinear Anal. 68(12), 3972–3986 (2008)

    Article  MathSciNet  MATH  Google Scholar 

  35. Musina, R., Nazarov, A.I.: Fractional Hardy–Sobolev inequalities on half spaces (2017). ArXiv e-prints

  36. Palatucci, G., Pisante, A.: Improved Sobolev embeddings, profile decomposition, and concentration-compactness for fractional Sobolev spaces. Calc. Var. Partial Differ. Equ. 50(3–4), 799–829 (2014)

    Article  MathSciNet  MATH  Google Scholar 

  37. Silvestre, L.: Regularity of the obstacle problem for a fractional power of the Laplace operator. Commun. Pure Appl. Math. 60(1), 67–112 (2007)

    Article  MathSciNet  MATH  Google Scholar 

  38. Tertikas, A., Tintarev, K.: On existence of minimizers for the Hardy–Sobolev–Maz’ya inequality. Ann. Mat. Pura Appl. (4) 186(4), 645–662 (2007)

    Article  MathSciNet  MATH  Google Scholar 

  39. Tzirakis, K.: Sharp trace Hardy–Sobolev inequalities and fractional Hardy–Sobolev inequalities. J. Funct. Anal. 270(12), 4513–4539 (2016)

    Article  MathSciNet  MATH  Google Scholar 

Download references

Acknowledgements

I would like to thank my Ph.D. supervisor Prof. K. Sandeep for countless valuable discussions and suggestions. Also, I would like to thank the anonymous referee for many useful suggestions.

Author information

Authors and Affiliations

Authors

Corresponding author

Correspondence to Arka Mallick.

Additional information

Communicated by A. Malchiodi.

Publisher's Note

Springer Nature remains neutral with regard to jurisdictional claims in published maps and institutional affiliations.

Appendix

Appendix

1.1 A density property

Main aim of this section is to derive Lemma 4.1. The arguments are modifications of those in [12], where the Muckenhoupt \(A_1\) properties of the weights have been used crucially. We will sketch the proof by pointing out main steps. First, let us define

$$\begin{aligned} \mathcal {W} := \left\{ u\in L^{2^*}\left( {\mathbb {R}}^{N};\frac{1}{|x'|^{{\tilde{\alpha }}2^*}}\right) : \int _{{\mathbb {R}}^{N}}\int _{{\mathbb {R}}^{N}}\frac{\left| u(x)-u(y) \right| ^2 dxdy}{\left| x-y \right| ^{N+2s}|x'|^{{\tilde{\alpha }}}|y'|^{{\tilde{\alpha }}}}< \infty \right\} \!, \end{aligned}$$

endowed with the following norm

$$\begin{aligned} \left| \left| u \right| \right| _{\mathcal {W}} := \left[ \left[ u\right] \right] _{s,{\tilde{\alpha }},{\mathbb {R}}^{N}} + \left| \left| u \right| \right| _{2^*,{\tilde{\alpha }},{\mathbb {R}}^{N}} . \end{aligned}$$

Here,

$$\begin{aligned} \left| \left| u \right| \right| _{2^*,{\tilde{\alpha }},{\mathbb {R}}^{N}} : = \left( \int _{{\mathbb {R}}^{N}} \frac{\left| u(x) \right| ^{2^*}}{|x'|^{{\tilde{\alpha }}2^*}}dx\right) ^{\frac{1}{2^*}}, \end{aligned}$$

and the semi-norm \(\left[ \left[ u\right] \right] _{s,{\tilde{\alpha }},{\mathbb {R}}^{N}}\) is same, as defined in Sect. 4. We also, define the following:

$$\begin{aligned} \text {when }&N' = 2N, \ w(z,z) = (z,z), \ \Theta (X) =|x'|^{{\tilde{\alpha }}}|y'|^{{\tilde{\alpha }}}, \ X=(x,y); \ x,y,z\in {\mathbb {R}}^{N}, \nonumber \\ \text {and when }&N' = N, \ w(z)= z,\ \Theta (X) = |x'|^{{\tilde{\alpha }}2^*},\ X=x ;\ x,y\in {\mathbb {R}}^{N}. \end{aligned}$$
(8.1)

Next, we will prove the following lemma.

Lemma 8.1

Let \(u\in C_c^\infty ({\mathbb {R}}^{N})\). Also, we consider \(\eta _\epsilon \), defined by the following

$$\begin{aligned} \eta _\epsilon (x')={\left\{ \begin{array}{ll} 0 , \text { if } |x'|<\epsilon ^2 \\ \frac{\ln \left( \frac{|x'|}{\epsilon ^2}\right) }{|\ln \epsilon |} , \text { if } \epsilon ^2\le |x'|\le \epsilon \\ 1 , \text { if } |x'|>\epsilon . \end{array}\right. } \end{aligned}$$

Then, for any \(0<{\tilde{\alpha }}\le (k-2s)/2\), the following are true

  1. (i)

    \(\int _{{\mathbb {R}}^{N}}\int _{{\mathbb {R}}^{N}} \frac{\left| u(x)-u(y) \right| ^2}{\left| x-y \right| ^{N+2s}|x'|^{{\tilde{\alpha }}}|y'|^{{\tilde{\alpha }}}} dxdy<\infty ,\)

  2. (ii)

    \(\displaystyle \lim _{\epsilon \rightarrow 0} \int _{{\mathbb {R}}^{N}}\int _{{\mathbb {R}}^{N}} \frac{u^2(x)\left| \eta _\epsilon (x')-\eta _\epsilon (y') \right| ^2}{\left| x-y \right| ^{N+2s}|x'|^{{\tilde{\alpha }}}|y'|^{{\tilde{\alpha }}}}dxdy =0\).

In particular, \(\eta _\epsilon u\in C_c^{0,1}({\mathbb {R}}^{N}_k)\) converges to u under the semi norm \(\left[ [.]\right] _{s,{\tilde{\alpha }},{\mathbb {R}}^{N}}\), i.e \(u\in \mathcal {{\dot{H}}}^{s,{\tilde{\alpha }}}({\mathbb {R}}^{N})\).

Proof

We will only prove (ii). One can easily check that (i) holds in fact for \(u\in C_c^{0,1}({\mathbb {R}}^{N})\). Notice that

$$\begin{aligned}&\int _{{\mathbb {R}}^{N}}\int _{{\mathbb {R}}^{N}} \frac{u^2(x)\left| \eta _\epsilon (x')-\eta _\epsilon (y') \right| ^2}{\left| x-y \right| ^{N+2s}|x'|^{{\tilde{\alpha }}}|y'|^{{\tilde{\alpha }}}}dxdy= \int _{{\mathbb {R}}^k}\int _{{\mathbb {R}}^k} \frac{\left| \eta _\epsilon (x')-\eta _\epsilon (y') \right| ^2}{|x'|^{{\tilde{\alpha }}}|y'|^{{\tilde{\alpha }}}}dx'dy' \nonumber \\&\int _{{\mathbb {R}}^{N-k}} u^2(x',x'') \int _{{\mathbb {R}}^{N-k}} \frac{dy''}{\left( |x'-y'|^2+|x''-y''|^2\right) ^{\frac{N+2s}{2}}} dx'' \nonumber \\&\quad \le C \int _{{\mathbb {R}}^k}\int _{{\mathbb {R}}^k} \frac{\left| \eta _\epsilon (x')-\eta _\epsilon (y') \right| ^2}{\left| x'-y' \right| ^{k+2s}|x'|^{{\tilde{\alpha }}}|y'|^{{\tilde{\alpha }}}}dx'dy', \end{aligned}$$
(8.2)

where (and for the rest of the proof) \(C>0\) is constant depending on \(N,k,s, {\tilde{\alpha }}\), \(\left| \left| u \right| \right| _{L^\infty ({\mathbb {R}}^{N})}\) and \({\text {supp}}u\). We define

$$\begin{aligned} I_\epsilon (x',y')&:= \frac{\left| \eta _\epsilon (x')-\eta _\epsilon (y') \right| ^2}{\left| x'-y' \right| ^{k+2s}|x'|^{{\tilde{\alpha }}}|y'|^{{\tilde{\alpha }}}}dx'dy' \text { and } H_\epsilon := \int _{{\mathbb {R}}^{N}}\int _{{\mathbb {R}}^{N}} I_\epsilon (x',y') dx'dy'. \end{aligned}$$

Then, in view of the (8.2), it is enough to show \(H_\epsilon = o(1)\) as \(\epsilon \rightarrow 0.\) We define

$$\begin{aligned} H_{\epsilon ,1}&:= \int _{|x'|<\epsilon ^2}\int _{\epsilon ^2<|y'|<\epsilon }I_\epsilon , \ H_{\epsilon ,2} : = \int _{|x'|>\epsilon }\int _{\epsilon ^2<|y'|<\epsilon } I_\epsilon , \\ H_{\epsilon ,3}&:= \int _{|x'|<\epsilon ^2} \int _{|y'|>\epsilon }, \ H_{\epsilon ,4} : = \int _{\epsilon ^2<|x'|<\epsilon }\int _{\epsilon ^2<|y'|<\epsilon } I_\epsilon . \end{aligned}$$

Then using the symmetry of \(I_\epsilon \) we have

$$\begin{aligned} H_\epsilon : = 2H_{\epsilon ,1}+ 2H_{\epsilon ,2}+2H_{\epsilon ,3} +H_{\epsilon ,4}. \end{aligned}$$

We will show \(H_{\epsilon ,m} = o(1)\) as \(\epsilon \rightarrow 0\) for \(m=1,2,3,4\).

Step 1 In this step will estimate \(H_{\epsilon ,1}.\) For this we define

$$\begin{aligned} F_{x'}:&=\{\epsilon ^2<|y'|<\epsilon \} \cap \{y': |y'-x'|\ge \epsilon ^2/2\} , \text { and } \\ F'_{x'} :&= \{\epsilon ^2<|y'|<\epsilon \} \cap \{y': |y'-x'|< \epsilon ^2/2\}. \text { Then }\\ H_{\epsilon ,1}&= \int _{|x'|<\epsilon ^2} \int _{F_{x'}}I_\epsilon + \int _{|x'|<\epsilon ^2}\int _{F'_{x'}} I_\epsilon . \end{aligned}$$

We first consider

$$\begin{aligned} \int _{|x'|<\epsilon ^2}\int _{F_{x'}'}I_\epsilon&\le \frac{C}{|\ln \epsilon |^2} \int _{|x'|<\epsilon ^2} \frac{1}{|x'|^{2{\tilde{\alpha }}}} \int _{F'_{x'}} \frac{\left| \ln |y'|-\ln |x'| \right| ^2}{|x'-y'|^{k+2s}} dy' dx' \nonumber \\&\le \int _0^1 \frac{C}{|\ln \epsilon |^2} \int _{|x'|<\epsilon ^2} \frac{1}{|x'|^{2{\tilde{\alpha }}}} \int _{F'_{x'}} \frac{dy'}{|x'-y'|^{k+2s-2}\left| y'+r(x'-y') \right| ^2} dx' dr \nonumber \\&\le \frac{C}{|\ln \epsilon |^2} \int _{|x'|<\epsilon ^2} \frac{1}{|x'|^{2{\tilde{\alpha }}}} \int _{F'_{x'}} \frac{dy'}{|x'-y'|^{k+2s-2}|y'|^2} dx' \nonumber \\&\le \frac{C}{\epsilon ^4|\ln \epsilon |^2} \int _{|x'|<\epsilon ^2} \frac{1}{|x'|^{2{\tilde{\alpha }}}} \int _{\{|x'-y'|<\frac{\epsilon ^2}{2}\}} \frac{dy'}{|x'-y'|^{k+2s-2}} dx' \nonumber \\&\le \frac{C}{\epsilon ^4|\ln \epsilon |^2} \epsilon ^{4s} \epsilon ^{4-4s} = o(1), \text { as } \epsilon \rightarrow 0 , \end{aligned}$$
(8.3)

where in the last inequality we have used the fact that, for small \(\epsilon >0\), \(\epsilon ^{k-2{\tilde{\alpha }}} \le \epsilon ^{4s}\), for any \(0<2{\tilde{\alpha }}\le k-2s\). Next, we consider

$$\begin{aligned} \int _{|x'|<\epsilon ^2} \int _{F_{x'}} I_\epsilon&\le \frac{C}{|\ln \epsilon |^2} \int _{|x'|\le \epsilon ^2}\frac{1}{|x'|^{{\tilde{\alpha }}}} \int _{F_{x'}} \frac{\ln ^2(\frac{|y'|}{\epsilon ^2})}{\left| x'-y' \right| ^{k+2s}|y'|^{{\tilde{\alpha }}}}dy' dx' \nonumber \\&\le \frac{C\epsilon ^{k-2s-2{\tilde{\alpha }}}}{|\ln \epsilon |^2} \int _{|x'|\le 1}\frac{1}{|x'|^{{\tilde{\alpha }}}} \int _{\begin{array}{c} \{1<|y'|<\frac{1}{\epsilon }\} \\ \cap \{|x'-y'|>\frac{1}{2}\} \end{array}} \frac{\ln ^2|y'|}{\left| x'-y' \right| ^{k+2s}|y'|^{{\tilde{\alpha }}}}dy' dx' \nonumber \\&\le o(1)+ \frac{C}{|\ln \epsilon |^2} \int _{|x'|<1} \int _{\begin{array}{c} \{2<|y'|<\frac{1}{\epsilon }\} \\ \cap \{|x'-y'|>\frac{1}{2}\} \end{array}}\frac{\ln ^2|y'|}{\left| x'-y' \right| ^{k+2s}|y'|^{{\tilde{\alpha }}}}dy' dx'\nonumber \\&\le o(1) +\frac{C}{|\ln \epsilon |^2}\int _{\{2<|y'|<\epsilon ^{-1}\}} \frac{\ln ^2|y'|dy'}{|y'|^{k+2s+\frac{k-2s}{2}}} \nonumber \\&\le o(1) +\frac{C}{|\ln \epsilon |^2}\int _{\{2<|y'|<\epsilon ^{-1}\}} \frac{\ln ^2|y'|dy'}{|y'|^{k+2s}} = o(1), \text { as } \epsilon \rightarrow 0. \end{aligned}$$
(8.4)

Hence, combining (8.3) and (8.4) we have \(H_{\epsilon ,1} = o(1),\) as \(\epsilon \rightarrow 0\).

Step 2 In this step, we will show that \(H_{\epsilon ,m} =o(1) \), as \(\epsilon \rightarrow 0\) for \(m=2,3\). In fact, we will show this, only for the case \(m=2\). The assertion, for the case, \(m=3\), will follow similarly and much more easily.

By a change of variable we get

$$\begin{aligned} H_{\epsilon ,2}&\le \frac{1}{|\ln \epsilon |^2} \int _{\epsilon<|y'|<1}\frac{1}{|y'|^{2{\tilde{\alpha }}}}\int _{|x'|>1} \frac{\left| \ln \frac{|y'|}{\epsilon }-\ln \frac{1}{\epsilon } \right| ^2}{|x'-y'|^{k+2s}} dy'dx' \nonumber \\&\le H'_{\epsilon ,2}+H''_{\epsilon ,2}, \end{aligned}$$
(8.5)

where

$$\begin{aligned} H'_{\epsilon ,2}&:= \frac{C}{|\ln \epsilon |^2} \int _0^1\int _{\epsilon<|y'|<1} \frac{1}{|y'|^{2{\tilde{\alpha }}}} \int _{\{|x'|>1\}\cap \{|x'-y'|\le \frac{1}{2}\}} \frac{dx'}{|x'-y'|^{k+2s-2}|x'+r(x'-y')|^2}dy'dr \\&\le \frac{C}{|\ln \epsilon |^2} \int _{\epsilon<|y'|<1} \frac{1}{|y'|^{2{\tilde{\alpha }}}}\int _{\{|x'-y'|\le \frac{1}{2}\}} \frac{dx'}{|x'-y'|^{k+2s-2}}dy' = o(1), \text { as } \epsilon \rightarrow 0, \end{aligned}$$

and

$$\begin{aligned} H''_{\epsilon ,2}&:= \frac{C}{|\ln \epsilon |^2} \int _{\epsilon<|y'|<1}\frac{1}{|y'|^{2{\tilde{\alpha }}}}\int _{\{|x'|>1\}\cap \{|x'-y'|\ge \frac{1}{2}\}} \frac{\left| \ln \frac{|y'|}{\epsilon }-\ln \frac{1}{\epsilon } \right| ^2}{|x'-y'|^{k+2s}} dy'dx' \\&\le \frac{C}{|\ln \epsilon |^2} \int _{\epsilon<|y'|<1} \frac{\ln ^2|y'|}{|y'|^{2{\tilde{\alpha }}}}dy' = \frac{C}{|\ln \epsilon |^2}\int _\epsilon ^1 \frac{\ln ^2r}{r^{1-\alpha '}} dr \\&= o(1), \text { as } \epsilon \rightarrow 0, \end{aligned}$$

where \(\alpha ' = k-2{\tilde{\alpha }}\ge 2s.\) Hence, from (8.5) we have \(H_{\epsilon ,2}= o(1),\) as \(\epsilon \rightarrow 0\).

Step 3 In this step, we will show that \(H_{\epsilon ,4} =o(1) \), as \(\epsilon \rightarrow 0\). Similarly, considering different regions, we see that, it is enough to show the following:

$$\begin{aligned} H_{\epsilon ,4,1}: = \frac{1}{|\ln \epsilon |^2} \int \int \limits _{F}\frac{\left| \ln |x'|-\ln |y'| \right| ^2}{\left| x'-y' \right| ^{N+2s}|x'|^{{\tilde{\alpha }}}|y'|^{{\tilde{\alpha }}}}dx'dy' = o(1), \text { as } \epsilon \rightarrow 0, \end{aligned}$$
(8.6)

where F is defined as follows:

$$\begin{aligned} F: = \{(x',y'): \epsilon ^2< |x'|\le |y'|<\epsilon \text { and } |y'|<2|x'|\}. \end{aligned}$$

Clearly, \(F\subset \{(x',y'): \epsilon ^2< |x'|\le |y'|<\epsilon \text { and } |x'- y'|<3|x'|\}\). So, using, \(\ln r \le r-1\), for \(r\ge 1\), we estimate

$$\begin{aligned} H_{\epsilon ,4,1}&\le \frac{1}{|\ln \epsilon |^2} \int _{\epsilon ^2<|x'|<\epsilon }\frac{1}{|x'|^{k-2s+2}} \int _{|x'-y'|<3|x'|} \frac{dy'}{|x'-y'|^{k+2s-2}}dx' \\&= \frac{C}{|\ln \epsilon |^2}\int _{\epsilon ^2<|x'|<\epsilon } \frac{dx'}{|x'|^k} = o(1), \text { as } \epsilon \rightarrow 0. \end{aligned}$$

Combining Step 1, Step 2 and Step 3 we conclude the lemma. \(\square \)

In light of the Lemma 8.1, it is enough to prove that \(C_c^\infty ({\mathbb {R}}^{N})\) is dense in \(\mathcal {W}\) to conclude Lemma 4.1. The following Lemma shows that, we can approximate \(u\in \mathcal {W}\) by a sequence of compactly supported functions lying in \(\mathcal {W}\).

Lemma 8.2

Let \(u\in \mathcal {W}\), \(0<{\tilde{\alpha }}\le \frac{k-2s}{2}\) and \(\eta \in C_c^\infty \left( B_2^N(0);[0,1]\right) \) such that \(\eta = 1\) in \(B_1^N(0)\) and \(\eta _j(x) = \eta (x/j)\). Then

$$\begin{aligned} \displaystyle {\lim _{j\rightarrow \infty }} \left[ \left| \left| u-\eta _ju \right| \right| _{2^*,{\tilde{\alpha }},{\mathbb {R}}^{N}} + \left[ \left[ u-\eta _ju\right] \right] _{s,{\tilde{\alpha }},{\mathbb {R}}^{N}}\right] =0. \end{aligned}$$

Proof

We define

$$\begin{aligned} I_j : = \int _{{\mathbb {R}}^{N}}\int _{{\mathbb {R}}^{N}\setminus B_j^N(0)} \frac{|u(y)|^2\left| \eta _j(x)-\eta _j(y) \right| ^2}{|x-y|^{N+2s}|x'|^{{\tilde{\alpha }}}|y'|^{{\tilde{\alpha }}}} dy dx. \end{aligned}$$

Since, \(\eta = 1\) on \(B_1^N(0),\) so, to prove the Lemma, it is enough to prove, \(\displaystyle {\lim _{j\rightarrow \infty }}I_j = 0\). We define the following sets

$$\begin{aligned} D_{j,0}&: = \{(x,y) \in {\mathbb {R}}^{N}\times ({\mathbb {R}}^{N}\setminus B_j^N(0)) : |x|\le |y|/2\},\\ D_{j,1}&: = \{(x,y) \in {\mathbb {R}}^{N}\times ({\mathbb {R}}^{N}\setminus B_j^N(0)) : |x|\ge |y|/2 \text { and } |x-y|\ge j\}, \\ D_{j,2}&: = \{(x,y) \in {\mathbb {R}}^{N}\times ({\mathbb {R}}^{N}\setminus B_j^N(0)) : |x|\ge |y|/2 \text { and } |x-y|\le j\}. \end{aligned}$$

For \(m=0,1,2\), we write

$$\begin{aligned} I_{j,m} : = \int \int _{D_{j,m}} \frac{|u(y)|^2\left| \eta _j(x)-\eta _j(y) \right| ^2}{|x-y|^{N+2s}|x'|^{{\tilde{\alpha }}}|y'|^{{\tilde{\alpha }}}} dy dx. \end{aligned}$$

Then

$$\begin{aligned} I_j = I_{j,0}+I_{j,1}+I_{j,2}. \end{aligned}$$
(8.7)

Now, we break

$$\begin{aligned} \frac{|u(y)|^2\left| \eta _j(x)-\eta _j(y) \right| ^2}{|x-y|^{N+2s}|x'|^{{\tilde{\alpha }}}|y'|^{{\tilde{\alpha }}}} = \frac{|\eta _j(x)-\eta _j(y)|^2}{|x-y|^{2s+2\sigma _m}} \frac{|u(y)|^2}{|x-y|^{N-2\sigma _m|x'|^{{\tilde{\alpha }}}|y'|^{{\tilde{\alpha }}}}}, \end{aligned}$$

where \(\sigma _0 = s,\) \(\sigma _1 \in (0,s)\) and \(s<\sigma _2<1\) such that \(\frac{N(N-2\sigma _2)}{N-2s}>\max \{N-k,k\}.\) We denote, \(\sigma '_m := \frac{N-2\sigma _2}{N-2s}\). Then using Hölder inequality

$$\begin{aligned} I_{j,m} \le \left( \int \int _{D_{j,m}} \frac{|\eta _j(x)-\eta _j(y)|^{\frac{N}{s}}}{\left| x-y \right| ^{N+\sigma _m\frac{N}{s}}}dxdy\right) ^{\frac{2s}{N}} \left( \int \int _{D_{j,m}} \frac{|u(y)|^{2^*} }{|x-y|^{N\sigma '_m }|x'|^{{\tilde{\alpha }}} |y'|^{{\tilde{\alpha }}}}dx dy \right) ^{\frac{N-2s}{N} }. \end{aligned}$$
(8.8)

Clearly,

$$\begin{aligned} \int \int _{D_{j,m}} \frac{|\eta _j(x)-\eta _j(y)|^{\frac{N}{s}}}{\left| x-y \right| ^{N+\sigma _m\frac{N}{s}}}dxdy \le j^{(s-\sigma _m)\frac{N}{s}} \int _{{\mathbb {R}}^{N}}\int _{{\mathbb {R}}^{N}} \frac{|\eta _j(x)-\eta _j(y)|^{\frac{N}{s}}}{\left| x-y \right| ^{N+\sigma _m\frac{N}{s}}}dxdy \le C j^{\frac{(s-\sigma _m)N}{s}}. \end{aligned}$$
(8.9)

Now, we consider

$$\begin{aligned} \int \int _{D_{j,0}} \frac{|u(y)|^{2^*} }{|x-y|^{N\sigma '_0 }|x'|^{{\tilde{\alpha }}} |y'|^{{\tilde{\alpha }}}}dx dy&\le \int _{|y|>j} \frac{|u(y)|^{2^*}}{|y'|^\frac{2^*{{\tilde{\alpha }}}}{2}} \int _{|x|<\frac{|y|}{2} } \frac{dx}{|x-y|^N|y'|^\frac{2^*{{\tilde{\alpha }}}}{2}} dy\nonumber \\&\le C\int _{|y|>j} \frac{|u(y)|^{2^*}}{|y'|^{\frac{2^*{{\tilde{\alpha }}}}{2}}|y|^{\frac{{{\tilde{\alpha }}}2^*}{2}}}dy \le C \int _{|y|>j} \frac{|u(y)|^{2^*}}{|y'|^{\alpha 2^*}}dy. \end{aligned}$$
(8.10)

Since, \(N\sigma '_1>N>N-k\), we estimate

$$\begin{aligned} \int \int _{D_{j,1}} \frac{|u(y)|^{2^*} }{|x-y|^{N\sigma '_1 }|x'|^{{\tilde{\alpha }}} |y'|^{{\tilde{\alpha }}}}dx dy&\le \int _{|y|>j} \int _{\begin{array}{c} \{|x-y|>j\} \\ \cap \{|x'|\le \frac{|y'|}{2} \} \end{array}} \frac{|u(y)|^{2^*} }{|x-y|^{N\sigma '_1}|x'|^{{\tilde{\alpha }}} |y'|^{{\tilde{\alpha }}}}dx dy\nonumber \\&\quad + \int _{|y|>j} \int _{\begin{array}{c} \{|x-y|>j\} \\ \cap \{|x'|\ge \frac{|y'|}{2}\} \end{array}}\frac{|u(y)|^{2^*} }{|x-y|^{N\sigma '_1 }|x'|^{{\tilde{\alpha }}} |y'|^{{\tilde{\alpha }}}}dx dy \nonumber \\&\le C \int _{|y|>j} \frac{|u(y)|^{2^*}}{|y'|^{\frac{{{\tilde{\alpha }}}2^*}{2}}} \int _{|x'|<\frac{|y'|}{2}} \frac{dx'}{|x'-y'|^{N\sigma '_1-N+k} |x'|^{\frac{{{\tilde{\alpha }}}2^*}{2}}} dy' \nonumber \\&\quad + C\int _{|y|>j}\frac{|u(y)|^{2^*}}{|y'|^{2^*{{\tilde{\alpha }}}}} \int _{|x-y|>j} \frac{dx}{|x-y|^{N\sigma '_1}} dy \nonumber \\&\le C\int _{|y|>j} \frac{|u(y)|^{2^*}}{|y'|^{{{\tilde{\alpha }}}2^*}} \frac{dy}{|y'|^{N\sigma '_1-N}} \nonumber \\&\quad + C\frac{1}{j^{N\sigma '_1-1}}\int _{|y|>j} \frac{|u(y)|^{2^*}}{|y'|^{2^*{{\tilde{\alpha }}}}}dy \nonumber \\&\le \frac{C}{j^{\frac{2N(s-\sigma _1)}{N-2s}}} \int _{|y|>j} \frac{|u(y)|^{2^*}}{|y'|^{2^*{{\tilde{\alpha }}}}}dy. \end{aligned}$$
(8.11)

Similarly, using \(N>N{\bar{\sigma }}_2> \max \{ N-k,k\}\), we can derive

$$\begin{aligned} \int \int _{D_{j,2}} \frac{|u(y)|^{2^*} }{|x-y|^{N\sigma '_1 }|x'|^{{\tilde{\alpha }}} |y'|^{{\tilde{\alpha }}}}dx dy \le \frac{C}{j^{\frac{2N(s-\sigma _2)}{N-2s}}} \int _{|y|>j} \frac{|u(y)|^{2^*}}{|y'|^{2^*{{\tilde{\alpha }}}}}dy. \end{aligned}$$
(8.12)

Hence, plugging (8.9), (8.10), (8.11) and (8.12) into (8.8) and then using (8.7) we get

$$\begin{aligned} I_j \le C \left| \left| u \right| \right| ^2_{L^{2^*}\left( {\mathbb {R}}^{N}\setminus B_j^N(0); \frac{1}{|x'|^{2{{\tilde{\alpha }}}}}\right) } \rightarrow 0 , \text { as } j \rightarrow \infty . \end{aligned}$$

This proves the lemma. \(\square \)

The next proposition is a reminiscence of the fact, that \(\Theta \) is in \(A_1\). Although, in this case, the proof is a direct consequence of Proposition 4.1 and 4.2 of [12].

Proposition 8.3

There exists a constant \(C>0\) such that for every \(X\in {\mathbb {R}}^{N}_k\times {\mathbb {R}}^{N}_k\), when \(N'=2N\) and \(X\in {\mathbb {R}}^{N}_k\), when \(N'=N\), the following inequality is true

$$\begin{aligned} \displaystyle {\sup _{r>0}}\frac{1}{r^N}\int _{B^N_r(0)}\frac{dz}{\Theta \left( X+w(z)\right) } \le \frac{C}{\Theta (X)}. \end{aligned}$$

Using Proposition 8.3 and the fact, that the measure \(\frac{dX}{\Theta (X)}\), is finite on compact sets of \({\mathbb {R}}^{N'}\), we can derive the following lemma which is related to the boundedness of the maximal operator.

Lemma 8.4

Let \(q>1\) and \(V:{\mathbb {R}}^{N'}\rightarrow {\mathbb {R}}\) be a measurable function. Then, for any \(r>0\),

$$\begin{aligned} \int _{{\mathbb {R}}^{N'}} \left( \frac{1}{r^n}\int _{B_r^{N'}(0)} \left| V(X-w(z)) \right| dz\right) ^q\frac{dX}{\Theta (X)} \le C \int _{{\mathbb {R}}^{N'}} \frac{|V(X)|^q}{\Theta (X)}, \end{aligned}$$

for some constant \(C>0\).

Next, for \(V:{\mathbb {R}}^{N'}:\rightarrow {\mathbb {R}}\) measurable, we define the following operator

$$\begin{aligned} V\star \eta _0 (X) := \int _{{\mathbb {R}}^{N}} V(X-w(z))\eta _0(z)dz, \end{aligned}$$

where \(\eta _0\) is a radially symmetric mollifier in \({\mathbb {R}}^{N}\), with \(\eta _0\ge 0\) and \({\text {supp}}\eta _0 \subset B_1^N(0)\). Notice that, when \(N'= N\), \(V\star \eta _0\) coincides with the usual convolution operator \(V*\eta _0\). As a consequence of Lemma 8.4, we could control appropriate weighted \(L^p\) norm of \(V\star \eta _0\). More precisely, we could derive the following proposition.

Proposition 8.5

There exists a constant \(C>0\), such that for any measurable function \(V: {\mathbb {R}}^{N'}\rightarrow {\mathbb {R}}\) we have

$$\begin{aligned} \int _{{\mathbb {R}}^{N'}} \left| V\star \eta _0 \right| ^p \frac{dX}{\Theta (X)} \le C\int _{{\mathbb {R}}^{N'}}|V(X)|^p\frac{dX}{\Theta (X)}, \end{aligned}$$

where \(p=2\), when \(N'=2N\) and \(p=2^*\), when \(N'=N\).

1.2 Proof of Lemma 4.1

Proof

We define

$$\begin{aligned} L^p({\mathbb {R}}^{N'};\Theta ) := \left\{ V:{\mathbb {R}}^{N'}\rightarrow {\mathbb {R}}\text { measurable }: \int _{{\mathbb {R}}^{N'}} |V(X)|^p \frac{dX}{\Theta (X)} <\infty \right\} , \end{aligned}$$

where p is defined in the Proposition 8.5. Then, since \(\frac{dX}{\Theta (X)}\) is finite on compact sets of \({\mathbb {R}}^{N'}\), so using Lusin’ s theorem and Proposition 8.5, we can prove that \(C_c^\infty ({\mathbb {R}}^{N'})\) is dense in \(L^p({\mathbb {R}}^{N};\Theta )\). As a consequence of this density and Proposition 8.2 and the fact, that for any \(u\in \mathcal {W}\) and \(\eta \in C_c^\infty ({\mathbb {R}}^{N})\), \(V^u*\eta = V^{u*\eta }\), we can prove that \(C_c^\infty ({\mathbb {R}}^{N})\) is dense in \(\mathcal {W}\), where \(V^u(x,y): = \frac{u(x)-u(y)}{\left| x-y \right| ^{\frac{N}{2}+s}}\), for \(x,y \in {\mathbb {R}}^{N}\). This proves that, \(\mathcal {{\dot{H}}}^{s,{\tilde{\alpha }}}({\mathbb {R}}^{N}) = \mathcal {W},\) which is exactly what we wanted to prove in Lemma 4.1. \(\square \)

Rights and permissions

Reprints and permissions

About this article

Check for updates. Verify currency and authenticity via CrossMark

Cite this article

Mallick, A. Extremals for fractional order Hardy–Sobolev–Maz’ya inequality. Calc. Var. 58, 45 (2019). https://doi.org/10.1007/s00526-019-1492-5

Download citation

  • Received:

  • Accepted:

  • Published:

  • DOI: https://doi.org/10.1007/s00526-019-1492-5

Mathematics Subject Classification

Navigation